LSAT and Law School Admissions Forum

Get expert LSAT preparation and law school admissions advice from PowerScore Test Preparation.

User avatar
 Dave Killoran
PowerScore Staff
  • PowerScore Staff
  • Posts: 5853
  • Joined: Mar 25, 2011
|
#41289
Complete Question Explanation
(The complete setup for this game can be found here: lsat/viewtopic.php?t=15690)

The correct answer choice is (C)

When S is third and T is fourth, J must be second (J cannot be fifth due to the last rule, and J cannot be first or last due to the fourth rule). Because H :longline: S, we can then deduce that H must speak first. We can then deduce that R must speak last (if R speaks fifth there would be a violation of the second rule). Finally, K must speak fifth as that is the only remaining slot:
J95_Game_#1_#3_diagram 1.png
Accordingly, answer choice (C) is correct.
You do not have the required permissions to view the files attached to this post.

Get the most out of your LSAT Prep Plus subscription.

Analyze and track your performance with our Testing and Analytics Package.